\documentclass[10pt,a4paper]{article} \usepackage[french]{babel} \usepackage[T1]{fontenc} \usepackage[utf8x]{inputenc} \usepackage{fancyhdr} \usepackage{amsfonts,amsmath,amssymb,mathrsfs,graphicx,amsthm,dsfont} \usepackage{euscript,mathrsfs} \usepackage{stmaryrd} \usepackage{comment} \usepackage{tikz} \setlength{\topmargin}{-25mm} \setlength{\textheight}{25.5cm} \setlength{\oddsidemargin}{-10mm} \setlength{\textwidth}{17.5cm} \newcommand{\htrait}{\smallskip\hspace{-0.6cm}\rule{20cm}{0.5pt}\smallskip} % Pour séparer les exos \newcount\sujetcount % compteur exo \sujetcount=0 \newcount\espcount % compteur qsp \espcount=0 \newcommand{\sujet}{\vspace{3mm}\par\advance\sujetcount by1 \begin{center} {\Large {\bf SUJET T\the\sujetcount}}\\ \vspace*{1cm} {\large {\bf Exercice principal T\the\sujetcount}} \end{center} \vspace*{2cm}} \newcommand{\Esp}{ \begin{center} {\Large {\bf Exercice sans préparation T\the\sujetcount}} \end{center} } \renewenvironment{comment}{\htrait \vspace*{4mm}\qquad \textbf{Solution :}\\ }{\par} % Pour masquer ou afficher les solutions \everymath{\displaystyle} % applique un displaystyle à toutes les commandes de math %ensembles de nombres \def\C{\mathbb{C}} \def\D{\mathbb{D}} \def\K{\mathbb{K}} \def\H{\mathbb{H}} \def\N{\mathbb{N}} \def\Z{\mathbb{Z}} \def\Q{\mathbb{Q}} \def\R{\mathbb{R}} \def\U{\mathbb{U}} \def\E{\mathbb{E}} \def\P{\mathbb{P}} \newcommand{\Nstar}{\ensuremath{\mathbb{N}^{*}}} \newcommand{\Zstar}{\ensuremath{\mathbb{Z}^{*}}} \newcommand{\Qstar}{\ensuremath{\mathbb{Q}^{*}}} \newcommand{\Rstar}{\ensuremath{\mathbb{R}^{*}}} \newcommand{\Cstar}{\ensuremath{\mathbb{C}^{*}}} \newcommand{\Rplus}{\ensuremath{\mathbb{R}_{+}}} \newcommand{\Rmoins}{\ensuremath{\mathbb{R}_{-}}} \newcommand{\Rplusstar}{\ensuremath{\mathbb{R}_{+}^*}} \newcommand{\Rmoinsstar}{\ensuremath{\mathbb{R}_{-}^*}} \newcommand{\Rplusetoile}{\ensuremath{\mathbb{R}_{+}^*}} \newcommand{\Rmoinsetoile}{\ensuremath{\mathbb{R}_{-}^*}} % Divers \newcommand{\e}{\mathrm{e}} %e de exponentielle \newcommand{\Id}{\hbox{\rm{Id}\,}} \newcommand{\1}[1]{\mathds{1}_{#1}}% la fonction indicatrice pour les proba \newcommand{\llb}{\llbracket} \newcommand{\rrb}{\rrbracket} %Relation de comparaison \renewcommand{\leq}{\leqslant} \renewcommand{\geq}{\geqslant} \newcommand{\peto}[1]{\rm{o}\left(#1\right)} \newcommand{\gdo}[1]{\rm{O}\left(#1\right)} \newcommand{\eq}[1]{\operatorname*{\sim}_{#1} } %Algèbre \def \M{{\cal M}} \newcommand{\Sp}{\text{Sp}\,} \newcommand{\Ker}{\mbox{ Ker }} \newcommand{\im}{\mbox{ Im }} \newcommand{\Vect}{\ensuremath{\mathop{\rm Vect\,}\nolimits}} \newcommand{\rg}{\ensuremath{\mathop{\rm rg\,}\nolimits}} \newcommand{\tr}{\ensuremath{\mathop{\rm Tr\,}\nolimits}} \newcommand{\abs}[1]{\left| #1 \right|}%valeur absolue \newcommand{\norme}[1]{\left|\left| #1 \right|\right|}%norme \newcommand{\prodscal}[2]{\langle #1 ; \; #2 \rangle} %produit scalaire %Mettre entre .. \newcommand{\pa}[1]{\ensuremath{\left(#1\right)}}%parenthèses \newcommand{\paf}[2]{\ensuremath{\left(\frac{#1}{#2}\right)}} % fractions entre parenthèses \newcommand{\acco}[1]{\ensuremath{\left\{ #1\right\}}}%accolades \newcommand{\crocint}[1]{\left[\begin{array}{c} #1 \end{array}\right]}% grands crochets pour les IPP % Intégrations \newcommand{\dx}{\ensuremath{\,\mathrm{d}x}} \newcommand{\dt}{\ensuremath{\,\mathrm{d}t}} \newcommand{\du}{\ensuremath{\,\mathrm{d}u}} \newcommand{\dv}{\ensuremath{\,\mathrm{d}v}} \newcommand{\dy}{\ensuremath{\,\mathrm{d}y}} % Numérotation des question \newcommand{\be}{\begin{enumerate}} \newcommand{\ee}{\end{enumerate}} \newcommand{\bi}{\begin{itemize}} \newcommand{\ei}{\end{itemize}} % Fonction \newcommand{\fonc}[4]{ \begin{array}{lcl} {#1} & \rightarrow & {#2} \\ {#3} & \mapsto & {#4} \\ \end{array} } \begin{document} \title{Rapport et sujets, oral HEC, Mathématiques (T) } \author{Juin-juillet 2021} \date{} \maketitle Le bilan de la session 2021 de mathématiques voie T est satisfaisant Le niveau des candidats est très hétérogène : les notes se sont étalées entre 4 et 19. La moyenne s'établit à 10,74 et l'écart-type à 4,59. On peut noter chez la plupart des candidats des lacunes en calculs. Mais fort heureusement, nombre d'entre eux les ont compensées par des raisonnements astucieux, un exposé clair et rigoureux et ont été récompensés par de bonnes, voire d'excellentes notes. Le jury aimerait insister sur les points suivants auprès des futur.e.s candidat.e.s et de leurs enseignant.e.s. \bi \item Les raisonnements graphiques et les tracés de courbes sont des compétences que nous souhaiterions fortement valoriser à l'avenir. Nous insistons sur le fait qu'après avoir tracé un tableau de variation, les candidats doivent être capable d'esquisser l'allure d'une courbe. \item Nous avons noté pas mal de lacunes en calcul, en particulier sur les calculs avec des puissances ou sur le calcul de dérivées. \item les théorèmes du cours doivent être connus précisément, avec leurs hypothèses précises et leurs conclusions. \item Nous avons aussi noté chez certains candidats quelques difficultés avec la formule des probabilités totales. \item Les questions informatiques ne doivent pas être négligées par les candidats. Elles ont permis à certains candidats d'améliorer significativement leur note. \item Tout n'est pas joué à l'issue de la préparation : au cours de la présentation de l'exercice préparé, le jury pose des questions pour aiguiller le candidat vers la solution. Il convient d'y être attentif. Il est souvent utile d'écrire au tableau ce qui est proposé par le jury. Par ailleurs, une prestation peut être jugée excellente sans que le candidats ne traite beaucoup de questions, alors qu'un candidat traitant de manière approximative un grand nombre de questions en déformant au passage les théorèmes de son cours risque d'être déçu par sa note finale. \item La question sans préparation est aussi très importante. Là encore, pour la plupart d'entre elles, le candidat peut tout à fait faire bonne impression sans aller au bout de la question. L'important est de réfléchir et d'écouter les indications du jury. Le candidat n'est pas obligé de parler instantanément en découvrant l'énoncé. Le jury a pu constater que certains candidats, qui avaient complètement raté l'exercice préparé ont redressé la barre sur l'exercice sans préparation, et parfois dans les toutes dernières minutes. \ei Voici quelques sujets proposés cette année. Nous publions aussi leurs corrigés, mais insistons sur le fait que ces corrigés sont indicatifs, ont été écrits à l'intention des membres du jury et ne correspondent pas toujours exactement à ce que l'on peut attendre des élèves. \newpage \sujet \noindent Une urne contient une boule blanche et deux boules noires. On effectue une succession de tirages avec remise de cette urne, jusqu'à ce que l'on ait obtenu au moins une fois une boule blanche et une boule noire. On note X le nombre de tirages nécessaires pour obtenir une boule blanche. On note $T$ la variable aléatoire désignant le nombre de tirages effectués. (On rappelle que les tirages s'arrêtent dès que l'on a obtenu une boule blanche et une boule noire) On notera aussi, pour $j\in\Nstar$, $N_j$ l'événement "le $j$-ième tirage donne une boule noire." \be \item {\bf Question de cours } : Loi géométrique de paramètre $p\in]0;1[$ : protocole, loi, espérance et variance. \item Déterminer la loi de X, son espérance et sa variance. \item \be\item Déterminer $T(\Omega)$ et calculer ${\mathbb P}([T=2])$. \item \'A l'aide du système complet $[N_1,\bar{N_1}]$ calculer ${\mathbb P}([T=k])$ pour $k\in T(\Omega)$. \ee \item La variable $e^X$ admet-elle une espérance ? On note $U$ la variable aléatoire prenant pour valeur le nombre de boules blanches au moment où l'on s'arrête. Par exemple, si les tirages ont donné successivement~: une boule noire, une boule noire,une boule noire et une boule blanche, alors $T=4$ et $U=1$. \item \be \item Déterminer $U(\Omega)$. \item Calculer ${\mathbb P}([U=1])$. Les variables U et T sont-elles indépendantes ? \ee \ee \begin{comment} \be \item Protocole ("premier succès" d'expériences identiques et indépendantes), $\forall k\in\Nstar$, $\P([X=k]]=(1-p)^{k-1}p$, $\E(X)=\frac1p$ et $V(X)=\frac{1-p}{p^2}$ (programme ECT2 page 5.) \item Les tirages sont identiques et indépendants. (avec remise) \fbox{$X\hookrightarrow{\mathcal G}\pa{\frac{1}{3}}$, $\E(X)=3$, $V(X)=6$.} \item \be\item Il faut au minimum deux tirages pour avoir une boule blanche et une boule noire. $T(\Omega)=\N\backslash\acco{1}$ $[T=2]=(B_1\cap N_2)\cup (N_1\cap B_2)$ Ces événements sont incompatibles : $\P([T=2])=\P(B_1\cap N_2)+ \P(N_1\cap B_2)$ Par indépendance : $\P([T=2])=\P(B_1)\P(N_2)+\P(N_1)\P(B_2)=\frac13\times \frac23+\frac23\times \frac13\boxed{=\frac49.}$ \item Avec la formule des probabilités totales, pour $k\geq 2$ : $\P([T=k])=\P([T=k]\cap N_1)+\P([T=k]\cap B_1)$ Si $k\geq 2$, $\P([T=k]\cap N_1)=\P(N_1\cap \cdots\cap N_{k-1}\cap B_k)$ et par indépendance : $\P([T=k]\cap N_1)=\paf{2}{3}^{k-1}\times \frac13$ On fait la même chose pour $\P([T=k]\cap B_1)$. $$\boxed{\P([T=k])=\frac13\paf23^{k-1}+\frac23\paf{1}{3}^{k-1}.}$$ \ee \item On utilise le théorème du transfert. Si cette série converge absolument, $\E(e^X)=\sum_{k=1}^{+\infty}\P([X=k])e^k=\sum_{k=1}^{+\infty}\frac13\times\paf23^{k-1}\times e^k$ La somme partielle donne pour $N\geq 0$, $S_N=\frac13\times e \times \sum_{k=0}^{N-1}\paf{2e}{3}^k$ Or, $\frac{2e}{3}>1$ donc la série diverge Ainsi \fbox{$e^X$ n'admet pas d'espérance. } \item \be\item On s'arrête lorsque après au moins une boule blanche. $U(\Omega)\subset\Nstar$. Et pour tout $k\in\Nstar$, $[U=k]$ est possible. $$\boxed{U(\Omega)=\N^*.}$$ \item Avec la formule des probabilités totales et le système complet $[N_1,B_1]$ : $\P([U=1]]=\P([U=1]\cap N_1)+\P([U=1]\cap B_1)$ $\P([U=1]\cap B_1)=\P(B_1\cap N_2)=\frac29$ $\P([U=1]\cap N_1)=\P(N_1)$ (si l'on a $N_1$, nécessairement $U=1)$ Ainsi \fbox{$\P([U=1])=\frac29+\frac23=\frac89$} * Le plus simple est de voir que si $T=2$, nécessairement, $U=1$, donc : $\P([U=1]|[T=2])=1\neq \P([U=1])$ \fbox{Les variables ne sont donc pas indépendantes.} \ee\ee \end{comment} \newpage \Esp Tracer le graphe de la fonction $x\mapsto \frac{1}{\ln(3-\abs{x})}$ \begin{comment} \be\item On commence par le domaine de définition. Il y a deux conditions : $3-\abs{x}>0$ et $\ln(3-\abs{x})\neq 0$ ssi $\abs{x}<3$ et $3-\abs{x}\neq 1$ ssi $\abs{x}\in[0;3[\backslash\acco{2}$ \fbox{$D_f=]-3;-2[\cup]-2;2[\cup]2;3[$} \item On peut noter que \fbox{$f$ est paire.} \item Si $x>0$ $\abs{x}=x$ Donc pour $x\in]{0};{2}[\cup]2;3[$, $f(x)=\frac{1}{\ln(3-x)}$. et $f'(x)=\frac{-1}{3-x}\times\frac{-1}{\ln^2(3-x)}=\frac{1}{(3-x)\ln^2(3-x)}>0$ \item On trace alors le tableau de variation que l'on complète avec les limites (pas de forme indéterminée) et qui permet de tracer la courbe $\begin{array}{|c|ccccccccccccccc|} \hline x&-3&&&&-2&&&0&&&2&&&&3\\ \hline &||&0&&&||&+\infty&&&&+\infty&||&&&0&||\\ f(x)&||&&\searrow&&||&&\searrow&&\nearrow&&||&&\nearrow&&||\\ &||&&&-\infty&||&&&\frac{1}{\ln(3)}&&&||&-\infty&&&||\\ \hline \end{array}$ {\it Note : le prolongement par continuité n'est pas au programme} \begin{tikzpicture}[scale=1] \clip (-3.5,-3) rectangle (3.5,3.3); \draw[thick, domain=-2.99999:-2.01, samples=200] plot(\x,{1/(ln(3+\x))}) ; \draw[thick, domain=-1.99:1.99, samples=200] plot(\x,{1/(ln(3-abs(\x)))}) ; \draw[thick, domain=2.01:2.9999, samples=200] plot(\x,{1/(ln(3-\x))}) ; %\draw[thick, domain=-2.99:-2.01, samples=80] plot(\x,{\x}) ; \draw [dashed](-3.5,0)--(3.5,0) (0,-3)--(0,3.3); \draw [dashed] (-2,-3)--(-2,3)(2,-3)--(2,3); \draw (-3,0) node[below right]{$-3$} node{$\bullet$}; \draw (3,0) node[below right]{$3$} node{$\bullet$}; \draw(0,{1/ln(3)}) node{$\bullet$}; \draw(2,0) node{$\bullet$}; \draw (0,1) node[above right]{$1$} node{$-$}; \draw (1,0) node[below right]{$1$} node{$|$}; \draw (0,0) node[below right]{$0$} node{$|$}; \draw (-1,0) node[below right]{$-1$} node{$|$}; \draw (-2,0) node[below right]{$-2$} node{$|$}; \draw (2,0) node[below right]{$2$} node{$|$}; \end{tikzpicture} \item Question supplémentaire soit $n\in\N$, déterminer le nombre de solution de l'équation $f(x)=n$. On énonce le théorème de la bijection L'équation admet deux solutions si $y\in\Rmoins^*\cup]{\frac{1}{\ln(3)}};{+\infty}[$ et une seule si $y=\frac{1}{\ln(3)}$ Sauf qu'on nous précise qu'ici, $n$ est un entier naturel. Comme $3>e$, $\ln(3)>\ln(e)=1$ et donc $\frac{1}{\ln(3)}\in]0;1[$ \fbox{Si $n\in\Nstar$, l'équation $f(x)=n$ admet deux solutions, l'équation $f(x)=0$ n'admet pas de solution} \ee \end{comment} \newpage \sujet \be\item {\bf Question de cours } : fonction continue en un point. Soit $n\in\N$. On définit la fonction $$f_n: \fonc{\R}{\R}{x}{ \left\{\begin{array}{cc} x(\ln(x))^n & \mbox{ si } x\in [1;2]\\ 0 & \mbox{ sinon } \end{array} \right.}$$ On donne l'approximation numérique suivante $\ln(2)\approx 0.7.$ \item Tracer l'allure de la courbe de $f_1$. Est-elle continue en tout point de $\R$ ? \item On note maintenant, pour $n\in\N$, $I_n=\int_1^2x.(\ln(x))^n\dx$. \be \item Calculer $I_0$. \item Montrer que la suite $(I_n)$ est décroissante. \item Montrer que $\lim_{n\to+\infty}I_n=0$. \ee \item \be \item Montrer que $\forall n\in\N$, $I_{n+1}=2(\ln(2))^{n+1}-\frac{n+1}{2}I_{n}$. \item Compléter le programme SCILAB de manière à afficher un entier N tel que : $\forall n\in\N$, si $n\geq N$, alors $I_n\leq \frac{1}{1000}$ \begin{verbatim} n=0; I= P= WHILE (I>0.001) P=P*log(2); I= end; disp( ); \end{verbatim} \ee \item Montrer qu'il existe un réel $C_n\in\R$ tel que la fonction $C_nf_n$ soit une densité de probabilité. \ee \begin{comment} \be\item $f$ est continue en un point $a$ de son domaine de définition ssi $\lim_{x\rightarrow a}f(x)=f(a)$. (programme ECT1 page 7.) \item On note que $f_1$ est dérivable sur $]1;2[$. $\forall x\in]1;2[$, $f'_1(x)=1+\ln(x)>1$. $f''_1(x)=\frac1x>0$ La fonction est convexe et croissante sur $[1;2]$, $f_1(1)=0$, $f'_1(1)=1$, $f_1(2)=2\ln(2)\approx 1,4$, $f''(2)=1+\ln(2)\approx 1,7$ On peut placer les demi-tangentes $f$ est continue en tout point de $\R$ sauf en 2. \begin{tikzpicture}[scale=1.5] \clip (-0.3,-0.3) rectangle (3.8,2); \draw[thick, domain=1:2, samples=80] plot (\x,{\x*ln(\x)}) ; \draw[thick, domain=-1:1, samples=80] plot (\x,{0}) ; \draw[thick, domain=2:4, samples=80] plot (\x,{0}) ; \draw [dashed](0,-0.3)--(0,2.2) (-0.3,0)--(2.2,0); \draw (1,0) node{$\bullet$}; \draw(2,{2*ln(2)}) node{$\bullet$}; \draw (1,0) node[below right]{$1$} node{$|$}; \draw (0,0) node[below right]{$0$} node{$|$}; \draw (3,0) node[below right]{$3$} node{$|$}; \draw (2,0) node[below right]{$2$} node{$|$}; \draw (0,1) node[below right]{$1$} node{$-$} ;\end{tikzpicture} \item \be\item $I_0=\int_1^2x\dx=\crocint{\frac{x^2}2}^2_1\boxed{=\frac32}$ \item Soit $n\in\Nstar$ $\forall x\in[1;2]$, $0\leq \ln(x)\leq \ln(2)\leq 1$, donc $(\ln(x))^{n+1}\leq (\ln(x))^n$ donc $x(\ln(x))^{n+1}\leq x(\ln(x))^n$ et par positivité de l'intégrale, $I_{n+1}\leq I_n$ \fbox{La suite $(I_n)$ est décroissante.} \item $\forall x\in[1;2]$, $0\leq x(\ln(x))^n\leq x(\ln(2))^n$ Par positivité de l'intégrale : $0\leq I_n\leq (\ln(2))^n\int_1^2x\dx=\frac{3(\ln(2))^n}{2}$ Comme $\abs{\ln(2)}<1$, $\lim_{n\to+\infty}(\ln(2))^n=0$ et donc par encadrement : \fbox{ $\lim_{n\to+\infty}I_n=0$} \ee \item \be\item On va faire une intégration par partie : $I_{n+1}=\int_1^2x(\ln(x))^{n+1}\dx$. On pose $u'(x)=x$, $u(x)=\frac{x^2}2$, $v(x)=(\ln(x))^{n+1}$, $v'(x)=\frac{(n+1)}{x}(\ln(x))^n$ Ainsi $I_{n+1}=\crocint{\frac{x^2.(\ln(x))^{n+1}}{2}}^2_1-\int_1^2\frac{x^2(n+1)(\ln(x))^n}{2x}\dx$ Donc \fbox{$I_{n+1}=2(\ln 2)^{n+1}-\frac{n+1}{2}I_n$} \item Comme la suite $(I_n$) est décroissante, il suffit de trouver N tel que $I_N\leq \frac{1}{1000}$. \begin{verbatim} n=0; I=1.5; P=1; WHILE (I>0.001) P=P*log(2); I=2*P-(n+1)*I/2; n=n+1; end; disp(n) \end{verbatim} \ee \item La fonction $C_nf_n$ est bien continue sauf en 2 (et admet des limites finies à gauche et à droite en ce point) Elle est bien positive si l'on choisit $C_n\geq 0$. De plus $\int_{-\infty}^{+\infty}C_nf_n(x)\dx=C_n\int_1^2x(\ln(x))^n\dx=C_n I_n$. Comme l'intégrale est strictement positive, on pose alors $C_n=\frac{1}{I_n}$, et l'on a bien \fbox{$C_nf_n$ est une densité de probabilité.} \ee \end{comment} \newpage \Esp Soient $p$ un réel de $]0;1[$, $X$ et $Y$ deux variables aléatoires indépendantes définies sur un espace probabilisé $(\Omega,{\mathcal A}, P)$. On suppose que $X(\Omega)=Y(\Omega)=\N$ et $\forall k\in\N$, $\P([X=k])=\P([Y=k])=\paf12^{k+1}$ On considère la matrice définie aléatoirement par $\forall \omega\in\Omega,\;\;M(\omega)=\begin{pmatrix}X(\omega)&Y(\omega)-3\\X(\omega)&3X(\omega)-2\end{pmatrix}$. Autrement dit $M=\begin{pmatrix}X&Y-3\\X&3X-2\end{pmatrix}$. Calculer la probabilité que cette matrice soit inversible. \begin{comment} \be\item La matrice est non inversible ssi $X(3X-2)-X(Y-3)=0$ ssi $X(3X-Y+1)=0$ ssi $X=0$ ou $Y=3X+1$. Or, $\P([X=0]\cup [Y=3X+1])=\P([X=0])+\P([Y=3X+1])-\P([Y=3X+1]\cap [X=0])$ $\P([X=0]\cup [Y=3X+1])=\frac12+\sum_{k=0}^{+\infty}\P([X=k]\cap [Y=3k+1])-\P([Y=3]\cap [X=0])$. $\P([X=0]\cup [Y=3X+1])=\frac12+\sum_{k=1}^{+\infty}\P([X=k]\cap [Y=3k+1])$. Or $\P([X=k]\cup [Y=3k+1])=\paf12^{k+1}\times \paf12^{3k+2}=\frac{1}{8}\times \paf{1}{16}^k$ et $\frac1{16}\in]0;1[$, la série converge. {\it Dans le programme officiel, ils n'ont pas la formule $\sum_{k=k_0}^{+\infty}q^k$} $\sum_{k=1}^{+\infty}\P([X=k]\cap [Y=3k+1]) =\frac{1}{8}\times \pa{\frac{1}{1-\frac{1}{16}}-1}= \frac{1}{8}\times \pa{\frac{16}{15}-1}=\frac{1}{8\times 15}=\frac{1}{120} $ Finalement \fbox{$\P([\mbox{M est inversible}])=1-\frac12-\frac{1}{120}=\frac{59}{120}$} \item Question supplémentaire : On suppose que $X=1$ et $Y=3$. Calculer alors $M^n$. $M=\begin{pmatrix}1&0\\1&1\end{pmatrix}$. Par récurrence ou binôme de Newton, \fbox{$M=\begin{pmatrix}1&0\\n&1\end{pmatrix}$} \ee \end{comment} \newpage \sujet Soient $\lambda\in\Rplusstar$ et $(X_n)_{n\in\N}$ une suite de variables aléatoires indépendantes suivant toutes une loi exponentielle de paramètre $\lambda$, définies sur un même espace probabilisé $(\Omega,{\mathcal A},\P)$. \be \item {\bf Question de cours } : loi exponentielle : densité, fonction de répartition, espérance et variance. \item On suppose, dans cette question seulement que $\lambda=1$. Tracer sur un même graphe une densité et la fonction de répartition de $X_1$. On note $Y=\exp(X_1)$ \item Sur quel intervalle Y prend-elle ses valeurs ? Déterminer la fonction de répartition de Y. \item On note pour $n\in\Nstar$, $M_n=\frac{X_1+\cdots +X_n}{n}=\frac{\sum_{k=1}^{n}X_k}{n}.$ \be\item Déterminer ${\mathbb E}(M_n).$ \item Montrer que $V(M_n)=\frac{1}{n\lambda^2}.$ \item En utilisant l'inégalité de Bienaymé Tchébytchev, montrer que $\forall n\in\Nstar$ et pour tout $a>0$ : $${\mathbb P}\pa{\left[-a< M_n-\frac{1}{\lambda}< a\right]}\geq 1-\frac{1}{n(a\lambda)^2}.$$ \ee \item Soit $\alpha\in]0;1[.$ \be \item Choisir $a>0$ tel que $\forall \lambda\geq 1$, $1-\frac{1}{n(a\lambda)^2}\geq 1-\frac{1}{na^2}\geq 1-\alpha.$ \ \\ \noindent On admet que l'on sait que $\lambda\geq 1$. \item Déduire de 4.c et de 5.a que l'intervalle $\left]M_n-\sqrt{\frac{1}{n\alpha}};M_n+\sqrt{\frac{1}{n\alpha}}\right[$ est un intervalle de confiance pour $\frac{1}{\lambda}$ au niveau de risque $\alpha$. \ee\ee \begin{comment} \be\item Une variable X suit une loi exponentielle de paramètre $\lambda$ ssi sa fonction de répartition vérifie $F(x)=0$ si $x\leq 0$ et $F(x)=1-\exp(-\lambda x)$ si $x>0$ Densité, $\E(X)=\frac{1}{\lambda}$, $V(X)=\frac{1}{\lambda^2}$ (programme ECT2 page 9.) \item $\forall x\in \Rplus$, $f(x)=\exp(-x)$ et $F(x)=1-\exp(-x)$ \begin{tikzpicture}[scale=1.5] \clip (-1,-0.3) rectangle (7,1.2); \draw[thick, domain=0:5.8, samples=80] plot (\x,{exp(-\x)}) ; \draw[domain=0:5.8, samples=80] plot (\x,{1-exp(-\x)}) ; \draw [dashed](0,0)--(5.8,0) (0,-2)--(0,1); \draw (-1,0)--(0,0); \draw (0,0) node{$\bullet$}; \draw(0,1) node{$\bullet$}; \draw (1,0) node[below right]{$1$} node{$|$}; \draw (0,0) node[below right]{$0$} node{$|$}; \draw (3,0) node[below right]{$3$} node{$|$}; \draw (2,0) node[below right]{$2$} node{$|$}; \end{tikzpicture} $F(0)=1$, $F_d'(0)=f(0)=1$, $f_d'(0)=-1$, on peut placer les tangentes. \item Comme $X_1$ prend ses valeurs dans $\Rplus$, \fbox{ Y prend ses valeurs dans $[1;+\infty[$} {\it la notation $X(\Omega)$ n'est pas dans le programme officiel} Si $x\leq 1$, $F_Y(x)=\P([Y\leq x])=0$, car l'événement $[Y\leq x]$ est impossible. Si $x>1$, $F_Y(x)=\P([Y\leq x])=\P(\exp(X_1)\leq x)=\P(X_1\leq \ln(x))$ (la fonction $\ln$ est strictement croissante) Ainsi \fbox{$F_Y(x)=F_X(\ln(x))=1-\exp(-\lambda \ln(x))=1-\frac{1}{x^\lambda}$} \item \be\item Par linéarité de l'espérance : $\E(M_n)=\frac{1}{n}\sum_{k=1}^n{n}\E(X_k)$\fbox{$=\frac{1}{\lambda}$} \item $V(M_n)=\frac{1}{n^2}V(\sum_{k=1}^nX_k)$ Comme les variables sont indépendantes : $V(M_n)=\frac{1}{n^2}\sum_{k=1}^nV(X_k)=\frac{1}{n^2}\sum_{k=1}^n\frac{1}{\lambda^2}=\frac{n}{n^2\lambda^2}$ \fbox{$=\frac{1}{n\lambda^2}$} \item On applique l'inégalité à la variable $M_n$, qui admet une espérance et une variance, et à $a>0$ $\P(\abs{M_n-\E(M_n)}\geq a)\leq \frac{V(M_n)}{a^2}$ On a $\E(M_n)=\frac{1}{\lambda}$ et $V(M_n)=\frac{1}{n\lambda^2}$ On passe à l'événement contraire : $\P(\abs{M_n-\E(M_n)}< a)\leq 1-\frac{V(M_n)}{a^2}$ Or, $\abs{M_n-\E(M_n)}< a$ ssi $-a\frac{1}{\lambda}>-\frac{1}{\sqrt{n\alpha}}+M_n $ ssi $\frac{1}{\lambda}\in \left]M_n-\sqrt{\frac{1}{n\alpha}};M_n+\sqrt{\frac{1}{n\alpha}}\right[$. \fbox{$\left]M_n-\sqrt{\frac{1}{n\alpha}};M_n+\sqrt{\frac{1}{n\alpha}}\right[$ est un intervalle de confiance pour $\frac{1}{\lambda}$ au niveau de risque $\alpha$.} \ee\ee \end{comment} \newpage \Esp Soit $q\in\Rplusstar$ fixé. On définit une suite $(u_n)$ par : $\forall n\in\N$, $u_n=\frac{q^n}{n!}$ Donner la valeur de $\lim_{n\rightarrow +\infty} u_n$ et prouvez le résultat proposé. \begin{comment} * Une réponse utilisant la convergence de la série exponentielle est bien sûr possible. * $\frac{u_{n+1}}{u_n}=\frac{q}{n+1}$, donc ($u_n>0)$ la suite est décroissante à partir du rang $N=\lfloor q \rfloor$ La suite $(u_n)$ est décroissante et minorée par 0, elle converge donc. Et si elle converge vers $\ell\neq 0$,par unicité de la limite $\frac{\ell}{\ell}=0$ C'est absurde Question supplémentaire : faire la preuve avec une autre méthode. * On peut aussi remarquer que pour $n\geq N$, $u_{n+1}\leq \paf{q}{N+1} u_n$ Par récurrence immédiate, $u_n\leq u_N\paf{q}{N+1}^{n-N}$ et comme $\frac{q}{N+1}\in]0;1[$, on peut conclure par encadrement. \end{comment} \end{document}